pharma q bank 80 p partv 2 23 Flashcards
(Question ID: 123) A 50-year-old man comes to the physician’s office expressing concern for his “ticker.” His father just died of a heart attack. The physicians had told his father that he had too much bad cholesterol. The patient wants to know how to reduce his risk of having a heart attack. His blood pressure is 150/95 mm Hg, and he is obese. He reports that he was taking some kind of statin before, but it made his legs cramp. The physician offers him an alternative drug to lower his bad cholesterol. This drug works by which of the following mechanisms?
a) Binding and excretion of bile-soluble acids
b) Decreased peripheral lipolysis
c) Inhibition of the rate-limiting enzyme of cholesterol formation
d) Retention of bile acid resins in hepatocytes
e) Suppressing liver gluconeogenesis
[Y]Binding and excretion of bile-soluble acids
(Question ID: 124) A 33-year-old woman presents to the emergency department with a 45-minute episode of sudden onset of palpitations. ECG demonstrates a regular narrow complex tachycardia. Upon discharge she is started on arrhythmic therapy with β blockers to suppress recurrences of supraventricular tachycardia. What β blocker used to treat arrhythmia prolongs the repolarization phase of the cardiac action potential?
a) Acebutolol
b) Esmolol
c) Metoprolol
d) Propranolol
e) Sotalol
[Y]Sotalol
(Question ID: 125) A 60-year-old man with Wolff-Parkinson-White syndrome currently receiving pharmacologic treatment presents to the clinic with fatigue and a 2.3-kg (5-lb) weight gain over the past 3 months. Physical examination results are normal. Laboratory studies show:
WBC count: 7000/mm3
Hematocrit: 44%
Hemoglobin: 15.6 g/dL
Platelet count: 270,000/mm3
Na+: 136 mEq/L
K+: 3.8 mEq/L
Cl ־:100 mEq/L
HCo3 ־: 24 mEq/L
Blood urea nitrogen: 12 mg/dL
Creatinine: 1.1 mg/dL
Thyroid-stimul3ting hormone: 23 U/mL
Thyroxine: 2.0 pg/dL
Prothrombin time: 12 seconds
Activated partial thromboplastin time: 30 seconds
The patient did not bring a list of his current medications.
a) Amiodarone
b) Enoxaparin
c) Metoprolol
d) Quinidine
e) Warfarin
[Y]Amiodarone
(Question ID: 126) A 35-year-old pregnant woman has severe, drug-resistant hypertension, aS well aS a history of cardiac dysrhythmias. Despite her cardiovascular conditions, the patient still has normal kidney function. An internist decides to try a potent vasodilator to control the patient’s hypertension. He also decides to co¬administer a β-blocker to prevent one of the more worrisome possible adverse effects of the medication. Which vasodilator was administered to the patient?
a) Amiodarone
b) Captopril
c) Hydralazine
d) Losartan
e) Methyldopa
[Y]Hydralazine
(Question ID: 127) An 82-year-old man is found unconscious in the park and is brought by ambulance to the nearest emergency department. Chart review reveals coronary artery disease, congestive heart failure, and moderate dementia; he has been taking digitalis for nearly 20 years. An ECG demonstrates atrial fibrillation Drug X is given, which successfully stops the atrial fibrillation, but 30 minutes later the patient exhibits nausea, vomiting, and atrial tachycardia with atrioventricular block. What is drug X?
a) Antidigitalis Fab
b) Lidocaine
c) Phenytoin
d) Quinidine
e) Tocainide
[Y]Quinidine
(Question ID: 128) A 45-year-old man with essential hypertension presents to the emergency department because of progressive muscle weakness. His blood pressure is 140/90 mm Hg, pulse is 85/min, respiratory rate is 13/min, and temperature is 37.C (98.6.F). ECG reveals peaked T waves and prolonged PR intervals. Which of the following medications could be the underlying cause of the changes noted on this patient’s ECG?
a) Acetazolamide
b) Furosemide
c) Hydrochlorothiazide
d) Mannitol
e) Spironolactone
[Y]Spironolactone
(Question ID: 129) A 52-year-old man is brought, unresponsive, to the emergency department by emergency medical services. A stat ECG shows ventricular fibrillation. Electrical defibrillation is initiated but is unsuccessful. The use of antiarrhythmic drugs to treat this shock-refractory ventricular fibrillation is discussed, but there are concerns about side effects of pulmonary fibrosis in this patient, who has a history of lung disease. What is the mechanism of action of the antiarrhythmic drug that was most likely administered?
a) Accelerates depolarization in high-frequency depolarizing cells
b) Prolongs depolarization via sodium channel blockade
c) Prolongs the repolarization phase via potassium channel blockade
d) Reduces calcium influx in depolarizing cells
e) Reduces sympathetic stimulation of myocardial cells
[Y]Prolongs the repolarization phase via potassium channel blockade
(Question ID: 130) A 45-year-old man presents to the clinic for an annual physical. He has a history of hypertension, and his medications include hydrochlorothiazide and metoprolol. At the clinic his blood pressure reading is 168/95 mm Hg. On a follow-up visit 3 weeks later his blood pressure remains elevated at 160/90 mm Hg. He is subsequently prescribed a transdermal medication to provide additional control of his hypertension. Which of the following describes the mechanism of action of this transdermal medication?
a) α1-Adrenergic receptor antagonist
b) α2-Adrenergic receptor agonist
c) β1-Adrenergic receptor antagonist
d) α2-Adrenergic receptor antagonist
e) Inhibitor of angiotensin-converting enzyme
[Y]α2-Adrenergic receptor agonist
(Question ID: 131) A patient presents to the emergency department with a severe headache, palpitations, and elevated blood pressure. He is found to have elevated urinary vanillylmandelic acid levels. He is diagnosed with a pheochromocytoma with predominantly elevated norepinephrine levels. Which of the following agents will antagonize both the vascular and cardiac actions of norepinephrine?
a) Atenolol
b) Doxazosin
c) Esmolol
d) Isoproterenol
e) Labetalol
[Y]Labetalol
(Question ID: 132) A 64-year-old man has a history of shortness of breath and edema in the lower extremities. One of the medications he takes is a sodium-potassium- chloride co־transport inhibitor. Which of the following is an indication for taking this type of medication?
a) Dilated cardiomyopathy
b) Hypertrophic obstructive cardiomyopathy
c) Hypokalemia
d) Hypotension
e) Primary hypoparathyroidism
[Y]Dilated cardiomyopathy
(Question ID: 133) A 54-year-old woman presents to the emergency department because of severe рain between her shoulder blades of 6 minutes’ duration. She states that she vomited on the way to the hospital. Her hands are clammy and she appears very anxious. An ECG from triage is shown in the image. The patient is given a medication that rapidly relieves her pain. What is the mechanism of action of this medication?
a) Inhibition of active chloride reabsorption at the early distal tubule via the sodium-chloride co־transporter
b) Inhibition of angiotensin-converting enzyme
c) Inhibition of the sodium/potassium adenosinetriphosphatase pump and increase in the atrioventricular node refractory period
d) Inhibition of transmembrane influx of calcium ions into myocardial cells
e) Suppression of prostaglandin and thromboxane production
f) Venodilation and coronary artery vasodilation
[Y]Venodilation and coronary artery vasodilation
(Question ID: 134) A 75-year-old man is admitted to the hospital for treatment of congestive heart failure. He has a known history of diabetes, Alzheimer’s disease, and high- altitude pulmonary edema, in addition to a severe allergy to all sulfa derivatives. Which of the following drugs would be appropriate for diuresis in this patient?
a) Acetazolamide
b) Ethacrynic acid
c) Furosemide
d) Hydrochlorothiazide
e) Mannitol
[Y]Ethacrynic acid
(Question ID: 135) A 62-year-old man who has been in the hospital for the past 4 days begins decompensating in the middle of the night. When examined, his blood pressure is 60/30 mm Hg, pulse is 112/min, and temperature is 40.6.C (105 ٠F). His breathing is rapid and shallow, and his arms and legs are warm to the touch. The patient is immediately given a 2-L bolus of intravenous fluids, with no resulting change in blood pressure. He is then admitted to the intensive care unit for suspected septic shock. Which of the following drugs should be used initially to stabilize the patient’s vital signs?
a) Clonidine
b) Metoprolol
c) Norepinephrine
d) Phenoxybenzamine
e) Vancomycin
[Y]Norepinephrine
(Question ID: 136) A 35-year-old woman presents for management of hypertension. She has tried for the past 6 months to manage her hypertension with diet and exercise, without success, and says that she is willing to try medication. The patient asks about trying timolol because her mother had success with it in treating her own hypertension. What missing piece of medical history might affect the choice of medical therapy for this patient?
a) Acute dissecting aortic aneurysm
b) Asthma
c) Glaucoma
d) Hypertension
e) Mild congestive heart failure
f) Myocardial infarction
[Y]Asthma
(Question ID: 137) An 80-year-old woman with a history of acute myocardial infarction and congestive heart failure (CHF) presents to the emergency department with a chief complaint of fevers and lethargy. After a full work-up, a urinalysis is performed, yielding a specimen with a pH of 5.5 and increased sodium, chloride, potassium, and calcium concentrations. Her CHF is managed with lisinopril, metoprolol, and a diuretic. Which portion of the renal tubule is mostly likely affected by this patient’s diuretic?
a) Collecting tubule
b) Distal convoluted tubule
c) Multiple portions
d) Proximal convoluted tubule
e) Thick ascending limb
[Y]Thick ascending limb
(Question ID: 138) A 57-year-old man with a medical history significant for angina and hypertension presents because of worsening angina. He treats each episode with sublingual nitroglycerin but has noticed that over the past year the frequency of “episodes” has increased from once a week to daily. The patient is currently taking lisinopril. A note in the chart indicates that the patient is allergic to metoprolol, so a calcium channel blocker is added to the patient’s regimen; this medication has a clinical effect most similar
a) Amlodipine
b) Diltiazem
c) Nifedipine
d) Nimodipine
e) Verapamil
[Y]Verapamil
(Question ID: 139) A 67-year-old woman presents to the emergency department complaining of dizziness. During the interview, she experiences two episodes of near-syncope. Physical examination reveals palpitations and slight bradycardia. Her daughter, who accompanies her, states that the patient is taking a medication for ”heart troubles” but says she cannot remember its name. Results of ECG are shown in the image. Which of the following medications is this patient likely taking that could both explain her symptoms and produce the abnormalities shown in this ECG?
a) Adenosine
b) Bretylium
c) Propranolol
d) Quinidine
e) Verapamil
[Y]Quinidine
(Question ID: 140) Calcium channel blockers commonly are used to treat hypertension, ischemic heart disease, and arrhythmias. Calcium channel blockers can be categorized aS dihydroperidines (including nifedipine) and non-dihydroperidines (including diltiazem). The different clinical uses of these drugs depend on their contrasting modes of action. The effectiveness of diltiazem in treating cardiac arrhythmias can be attributed to what mechanism?
a) Increase in cardiac chronotropism
b) Increase in conduction velocity
c) Increase in vascular smooth muscle tone
d) Prolongation of the PR interval
e) Prolongation of the QT interval
[Y]Prolongation of the PR interval
(Question ID: 141) A 48-year-old woman with a history of hypercholesterolemia and smoking presents to the cardiologist complaining of activity-induced chest pain. She describes the pain aS a chest tightness that develops after walking several blocks and goes away after several minutes of rest. In addition to arranging further testing, the physician gives the patient a tablet and tells her to put it under her tongue when she has any similar episodes. At low doses, which of the following is the primary mechanism by which this medication alleviates the patient’s symptoms?
a) Decrease platelet activation in response to injury
b) Decreases afterload, decreases myocardial oxygen demands
c) Decreases plaque formation on coronary arteries
d) Decreases preload, decreases myocardial oxygen demands
e) Increases afterload and decreases myocardial oxygen demands
f) Increases preload, decreases myocardial oxygen demands
[Y]Decreases preload, decreases myocardial oxygen demands
(Question ID: 142) A 54-year-old contractor complains of anginal pain that occurs at rest. On examination, his blood pressure is 145/90 and his heart rate is 90. A treatment of angina that often decreases the heart rate and can prevent vasospastic angina attacks is
a) Diltiazem
b) Nifedipine
c) Nitroglycerin
d) Propranolol
e) Timolol
[Y]Diltiazem
(Question ID: 143) In a patient receiving digoxin for congestive heart failure, conditions that may facilitate the appearance of toxicity include
a) Hyperkalemia
b) Hypernatremia
c) Hypocalcemia
d) Hypomagnesemia
e) Hypophosphatemia
[Y]Hypomagnesemia
(Question ID: 144) A 29-year-old accountant has recurrent episodes of tachycardia that sometimes convert to sinus rhythm spontaneously but more often require medical treatment. A drug that is commonly given as an intravenous bolus for the purpose of converting AV nodal tachycardias to normal sinus rhythm is
a) Adenosine
b) Amiodarone
c) Lidocaine
d) Quinidine
e) Sotalol
[Y]Amiodarone
(Question ID: 145) A 67 year old man comes to the office because he is having difficulty maintaining an erection. He has a past medical history significant for hypertension, hyperlipidemia and diabetes mellitus type II. He notes that about once a month he will have chest pain on exertion that lasts for only a few minutes once he rests, but he denies any chest pain while at rest. He smokes one pack of cigarettes daily and drinks two to three beers a week. The physician prescribes him a medication for his erectile dysfunction but is concerned about interaction with his other medications. Taking which of the following cardiac medications would be contraindicated?
a) Hydrochlorothiazide
b) Lisinopril
c) Metiprolol
d) Nitrate
e) Simvastatine
[Y]Nitrate
(Question ID: 149) A 57-year-old man presents to his physician with periorbital swelling and edema of the lips and tongue. Two weeks prior to presentation, the patient was hospitalized with a ST segment elevation myocardial infarction. His blood pressure at the hospital was noted to be 174/103. The patient was discharged from the hospital and placed on several medications including aspirin; propanolol, lisinopril, isosorbide dinitrate, and hydrochlorothiazide. Which of the following is the most likely cause of this patient’s symptoms?
a) Aspirin
b) Hydrochlorothiazide
c) lsosorbide dinitrate
d) Lisinopril
e) Propranolol
[Y]Lisinopril
(Question ID: 150) A 69-year-old woman comes to the hospital with shortness of breath that is worst when she lays down in bed. Her past medical history is significant for hypertension, coronary artery disease; and diabetes mellitus. She also had an inferior wall myocardial infarction two years ago. Her last echocardiogram three months ago showed impaired systolic function with a left ventricular ejection fraction of 30%. Her blood pressure is 125/67, heart rate 73 beats per minute, and respiratory rate of 23 respirations per minute. Pulse oximetry shows oxygen saturation of 92% on room air. On examination she has crackles throughout her lung fields. Her cardiac examination reveals a normal S1 and S2 with no evidence of murmur. Chest x-ray shows bilateral pulmonary infiltrates consistent with pulmonary edema. Which of the following medications should be started to improve the patient’s shortness of breath?
a) Carvedilol
b) Furosemide
c) Lisinopril
d) Metoprolol
e) Spironolactone
[Y]Furosemide
(Question ID: 320) A 55-year-old woman with hypertension is to be treated with a vasodilator drug. Drugs X and Y have the same mechanism of action. Drug X in a dose of 5 mg produces the same decrease in blood pressure as 500 mg of drug Y. Which of the following statements best describes these results?
a) Drug Y is less efficacious than drug X
b) Drug X is about 100 times more potent than drug Y
c) Toxicity of drug X is less than that of drug Y
d) Drug X has a wider therapeutic window than drug Y
e) Drug X will have a shorter duration of action than drug Y because less of drug X is present over the time course of drug action
[Y]Drug X is about 100 times more potent than drug Y
(Question ID: 321) Sugammadex is a new drug that reverses the action of rocuronium and certain other skeletal muscle-relaxing agents. It appears to interact directly with the rocuronium molecule and not at all with the rocuronium receptor. Which of the following terms best describes sugammadex?
a) Chemical antagonist
b) Noncompetitive antagonist
c) Partial agonist
d) Pharmacologic antagonist
e) Physiologic antagonist
[Y]Chemical antagonist
(Question ID: 322) A study was carried out in isolated, perfused animal hearts. In the absence of other drugs, pindolol, a β-adrenoceptor ligand, caused an increase in heart rate. In the presence of highly effective β stimulants, however, pindolol caused a dose-dependent, reversible decrease in heart rate. Which of the following expressions best describes pindolol?
a) A chemical antagonist
b) An irreversible antagonist
c) A partial agonist
d) A physiologic antagonist
e) A spare receptor agonist
[Y]A partial agonist
(Question ID: 323) Verapamil and phenytoin are both eliminated from the body by metabolism in the liver. Verapamil has a clearance of 1.5 L/min, approximately equal to liver blood flow, whereas phenytoin has a clearance of 0.1 L/min. When these compounds are administered along with rifampin, a drug that markedly increases hepatic drug-metabolizing enzymes, which of the following is most likely?
a) The half-lives of both verapamil and phenytoin will be markedly increased
b) The clearance of both verapamil and phenytoin will be markedly decreased
c) The clearance of verapamil will be unchanged, whereas the clearance of phenytoin will be increased
d) The half-life of phenytoin will be unchanged, whereas the half-life of verapamil will be increased
e) The clearance of both drugs will be unchanged
[Y]The clearance of verapamil will be unchanged, whereas the clearance of phenytoin will be increased
(Question ID: 324) A 63-year-old woman in the intensive care unit requires an infusion of procainamide. Its half-life is 2 h. The infusion is begun at 9 AM. At 1 PM on the same day, a blood sample is taken; the drug concentration is found to be 3 mg/L. What is the probable steady-state drug concentration, for example, after 12 or more hours of infusion?
a) 3 mg/L
b) 4 mg/L
c) 6 mg/L
d) 9.9 mg/L
e) 15 mg/L
[Y]4 mg/L
(Question ID: 325) A 30-year-old man is brought to the emergency department in a deep coma. Respiration is severely depressed and he has pinpoint pupils. His friends state that he self-administered a large dose of morphine 6 h earlier. An immediate blood analysis shows a morphine blood level of 0.25 mg/L. Assuming that the Vd of morphine in this patient is 200 L and the half-life is 3 h, how much morphine did the patient inject 6 h earlier?
a) 25 mg
b) 50 mg
c) 100 mg
d) 200 mg
[Y]200 mg
(Question ID: 326) Gentamicin, an aminoglycoside antibiotic, is sometimes given in intermittent intravenous bolus doses of 100 mg 3 times a day to achieve target peak plasma concentrations of about 5 mg/L. Gentamicin’s clearance (normally 5.4 L/h/70 kg) is almost entirely by glomerular filtration. Your patient, however, is found to have a creatinine clearance one third of normal. What should your modified dosage regimen for this patient be?
a) 20 mg 3 times a day
b) 33 mg 3 times a day
c) 72 mg 3 times a day
d) 100 mg 2 times a day
e) 150 mg 2 times a day
[Y]33 mg 3 times a day
(Question ID: 327) A 3-year-old child has swallowed the contents of 2 bottles of a nasal decongestant whose primary ingredient is a potent, selective α-adrenoceptor agonist drug. Which of the following is a sign of α-receptor activation that may occur in this patient?
a) Bronchodilation
b) Cardiac acceleration (tachycardia)
c) Pupillary dilation (mydriasis)
d) Renin release from the kidneys
e) Vasodilation of the splanchnic vessels
[Y]Pupillary dilation (mydriasis)
(Question ID: 328) Full activation of the sympathetic nervous system, as in the fight-or-flight reaction, may occur during maximal exercise. Which of the following effects is likely to occur?
a) Bronchoconstriction
b) Increased intestinal motility
c) Decreased renal blood flow
d) Miosis
e) Decreased heart rate (bradycardia)
[Y]Decreased renal blood flow
(Question ID: 329) A 30-year-old woman undergoes abdominal surgery. In spite of minimal tissue damage, complete ileus (absence of bowel motility) follows, and she complains of severe bloating. She also finds it difficult to urinate. Mild cholinomimetic stimulation with bethanechol or neostigmine is often effective in relieving these complications of surgery. Neostigmine and bethanechol in moderate doses have significantly different effects on which one of the following?
a) Gastric secretion
b) Neuromuscular end plate
c) Salivary glands
d) Sweat glands
e) Ureteral tone
[Y]Neuromuscular end plate
(Question ID: 330) A 63-year-old women has been treated for myasthenia gravis for several years. She reports to the emergency department complaining of recent onset of weakness of her hands, diplopia, and difficulty swallowing. She may be suffering from a change in response to her myasthenia therapy, that is, a cholinergic or a myasthenic crisis. Which of the following is the best drug for distinguishing between myasthenic crisis (insufficient therapy) and cholinergic crisis (excessive therapy)?
a) Atropine
b) Edrophonium
c) Physostigmine
d) Pralidoxime
e) Pyridostigmine
[Y]Edrophonium
(Question ID: 331) A 63-year-old man has just been diagnosed with dysautonomia (chronic idiopathic autonomic insufficiency). You are considering different therapies for his disease. Pyridostigmine and neostigmine may cause which one of the following?
a) Bronchodilation
b) Cycloplegia
c) Diarrhea
d) Irreversible inhibition of acetylcholinesterase
e) Reduced gastric acid secretion
[Y]Diarrhea
(Question ID: 332) Actions and clinical uses of muscarinic cholinoceptor agonists include which one of the following?
a) Bronchodilation (asthma)
b) Improved aqueous humor drainage (glaucoma)
c) Decreased gastrointestinal motility (diarrhea)
d) Decreased neuromuscular transmission and relaxation of skeletal muscle (during surgical anesthesia)
e) Increased sweating (fever)
[Y]Improved aqueous humor drainage (glaucoma
(Question ID: 333) A 3-year-old child is admitted after taking a drug from her parents’ medicine cabinet. The signs suggest that the drug is an indirect-acting cholinomimetic with little or no CNS effect and a duration of action of about 2–4 h. Which of the following is the most likely cause of these effects?
a) Acetylcholine
b) Bethanechol
c) Neostigmine
d) Physostigmine
e) Pilocarpine
[Y]Neostigmine
(Question ID: 334) Which of the following is an expected effect of a therapeutic dose of an antimuscarinic drug?
a) Decreased cAMP (cyclic adenosine monophosphate) in cardiac muscle
b) Decreased DAG (diacylglycerol) in salivary gland tissue
c) Increased IP3 (inositol trisphosphate) in intestinal smooth muscle
d) Increased potassium efflux from smooth muscle
e) Increased sodium influx into the skeletal muscle end plate
[Y]Decreased DAG (diacylglycerol) in salivary gland tissue
(Question ID: 335) A 65-year-old woman with long-standing diabetes mellitus is admitted to the ward from the emergency department, and you wish to examine her retinas for possible changes. Which of the following drugs is a good choice when pupillary dilation—but not cycloplegia—is desired?
a) Isoproterenol
b) Norepinephrine
c) Phenylephrine
d) Pilocarpine
e) Tropicamide
[Y]Phenylephrine
(Question ID: 336) A 30-year-old man is admitted to the emergency department after taking a suicidal overdose of reserpine. His blood pressure is 50/0 mm Hg and heart rate is 40 bpm. Which of the following would be the most effective cardiovascular stimulant?
a) Amphetamine
b) Clonidine
c) Cocaine
d) Norepinephrine
e) Tyramine
[Y]Norepinephrine
(Question ID: 337) A 56-year-old man has hypertension and an enlarged prostate, which biopsy shows to be benign prostatic hyperplasia. He complains of urinary retention. Which of the following drugs would be the most appropriate initial therapy?
a) Albuterol
b) Atenolol
c) Metoprolol
d) Prazosin
e) Timolol
[Y]Prazosin
(Question ID: 338) Which of the following best describes the mechanism of action of benzodiazepines?
a) Activate GABA-B receptors in the spinal cord
b) Block glutamate receptors in hierarchical neuronal pathways in the brain
c) Increase frequency of opening of chloride ion channels coupled to GABA-A receptors
d) Inhibit GABA transaminase to increase brain levels of GABA
e) Stimulate release of GABA from nerve endings in the brain
[Y]Increase frequency of opening of chloride ion channels coupled to GABA-A receptors
(Question ID: 339) Which drug used in the maintenance treatment of patients with tonic-clonic or partial seizure states increases the hepatic metabolism of many drugs including both warfarin and phenytoin?
a) Buspirone
b) Chlordiazepoxide
c) Eszopiclone
d) Phenobarbital
e) Triazolam
[Y]Phenobarbital
(Question ID: 340) A young male patient suffers from a seizure disorder characterized by tonic rigidity of the extremities followed in 15–30 s of tremor progressing to massive jerking of the body. This clonic phase lasts for 1 or 2 min, leaving the patient in a stuporous state. Of the following drugs, which is most suitable for longterm management of this patient?
a) Carbamazepine
b) Clonazepam
c) Ethosuximide
d) Felbamate
e) Tiagabine
[Y]Carbamazepine
(Question ID: 341) A 30-year-old male patient is on drug therapy for a psychiatric problem. He complains that he feels “flat” and that he gets confused at times. He has been gaining weight and has lost his sex drive. As he moves his hands, you notice a slight tremor. He tells you that since he has been on medication he is always thirsty and frequently has to urinate. The drug he is most likely to be taking is
a) Carbamazepine
b) Clozapine
c) Fluphenazine
d) Lithium
e) Valproic acid
[Y]Lithium
(Question ID: 342) Which of the following drugs has a high affinity for 5-HT2 receptors in the brain, does not cause extrapyramidal dysfunction or hematotoxicity, and is reported to increase the risk of significant QT prolongation?
a) Clozapine
b) Haloperidol
c) Olanzapine
d) Ziprasidone
[Y]Ziprasidone